Please confirm topic selection

Are you sure you want to trigger topic in your Anconeus AI algorithm?

Please confirm action

You are done for today with this topic.

Would you like to start learning session with this topic items scheduled for future?

Review Question - QID 213374

QID 213374 (Type "213374" in App Search)
A 33-year-old man presents to the emergency department with severe anxiety. He has had multiple episodes in the past treated with low dose lorazepam. The patient states that he feels as if he is going to die and that he cannot breathe. His past medical history is notable for depression and anxiety. His temperature is 98.1°F (36.7°C), blood pressure is 122/83 mmHg, pulse is 153/min, respirations are 13/min, and oxygen saturation is 98% on room air. The patient is given a low dose of lorazepam and reports a complete resolution of his symptoms. An ECG is performed and demonstrates prolongation of the P-R interval with a widened QRS complex. There is a P wave preceding every QRS complex, no dropped QRS complexes, and the P-R interval does not change. His initial lab values are unremarkable. Which of the following is the best management of this patient?